Probability of a pair of dice landing on a 4 and on a 6

Answers

Answer 1

Answer:

1/3

Step-by-step explanation:

It would be 2 options out of 6 total which means 2/6. 2/6 reduces down to 1/3


Related Questions

Enter the y coordinate of the solution to this system of equations. 3x+y=-2 x-2y=4

Answers

The y coordinate of the solution to this system of equations is -2

Calculating the y coordinate of the solution to this system of equations.

From the question, we have the following parameters that can be used in our computation:

3x+y=-2 x-2y=4

Express properly

So, we have

3x + y = -2

x - 2y = 4

Multiply the second equation by -3

so, we have the following representation

3x + y = -2

-3x + 6y = -12

Add the equations to eliminate x

7y = -14

Divide both sides by 7

y = -2

Hence, the value of y is -2

Read more about system of equations at

https://brainly.com/question/13729904

#SPJ1

Find two acute angles that satisfy the equation sin(3x + 9) = cos(x + 5). check that your answers make sense.

Answers

The equation sin(3x + 9) = cos(x + 5) has no solutions in the set of acute angles.

What are the acute angles that satisfy sin(3x + 9) = cos(x + 5)?

To find two acute angles that satisfy the equation sin(3x + 9) = cos(x + 5), we can use the trigonometric identity cos(x) = sin(π/2 - x) to rewrite the right-hand side of the equation as follows:

sin(3x + 9) = cos(x + 5)sin(3x + 9) = sin(π/2 - x - 5)3x + 9 = π/2 - x - 5 + 2πn or 3x + 9 = x + 5 + 2πn + π (where n is an integer)4x = -4 - 2πn or 2x = -2πn - 4 or 2x = π - 2πn - 4

Dividing both sides of the equation by 4, we get:

x = -(1/2)πn - 1

So the solutions are given by:

x = -(1/2)π - 1 and x = -(3/2)π - 1

To check that these solutions make sense, we need to ensure that they are acute angles, i.e., angles that measure less than 90 degrees.

The first solution, x = -(1/2)π - 1, can be written in degrees as:

x ≈ -106.26 degrees

This angle is not acute, so it is not a valid solution.

The second solution, x = -(3/2)π - 1, can be written in degrees as:

x ≈ -286.87 degrees

This angle is also not acute, so it is not a valid solution.

Therefore, there are no acute angles that satisfy the equation sin(3x + 9) = cos(x + 5).

Learn more about acute angles

brainly.com/question/10334248

#SPJ11

Of students taking both English 12 Honors and a senior level math course (AP Stats, AP Calculus, Pre-Calculus, College Prep Math, or Topics), 37% of students got an A in English, and 24% of students got an A in Math. 16% got an A in both classes.



What is the probability that a randomly selected student got an A in Math, but not English?

Answers

The probability that a randomly selected student got an A in Math, but not English, is 8%

Let A be the event that a student got an A in Math, and B be the event that a student got an A in English. Then, we want to find P(A and not B), or the probability that a student got an A in Math, but not English.

We know that P(A and B) = 0.16, or the probability that a student got an A in both Math and English. We also know that P(B) = 0.37, or the probability that a student got an A in English. Therefore, the probability of a student getting an A in Math, given that they got an A in English, can be calculated using the formula for conditional probability:

P(A | B) = P(A and B) / P(B)

P(A | B) = 0.16 / 0.37

P(A | B) = 0.43

This means that the probability of a student getting an A in Math, given that they got an A in English, is approximately 0.43.

To find the probability of a student getting an A in Math, but not English, we can subtract the probability of getting an A in both classes from the probability of getting an A in Math:

P(A and not B) = P(A) - P(A and B)

P(A and not B) = 0.24 - 0.16

P(A and not B) = 0.08

Therefore, the probability that a randomly selected student got an A in Math, but not English, is 0.08 or 8%.

To learn more about probability : https://brainly.com/question/251701

#SPJ11

When do you hit the water and what is your maximum height above the pool?

Answers

The time till you hit the water, given your height above the water, would be 0.76 seconds.

The maximum height above the pool you would get is 15.39 feet.

How to find the maximum height and time ?

We are given h ( t ) = - 16 t ² + 5t + 15

You hit the water at 0 so the formula is:

0 = - 16 t ² + 5t + 15

Using the quadratic equation, we can solve:

t = ( - b ± √ ( b ² - 4  ac ) ) / 2a

t = ( - 5 ± √ ( 5 ² - 4 ( - 16 ) ( 15 ) )) / 2 (- 16)

t = (- 5 ± √ 985 ) / -32

t = 0. 76 seconds

The vertex of the parabolic function would be:

= - b / 2a

= - 5 / ( 2 x - 16 )

=  0. 15625 seconds

Maximum height is therefore:

= -16 ( 0. 15625 ) ² + 5 ( 0. 15625 ) + 15

=  15.39 feet

Find out more on maximum height at https://brainly.com/question/23652112


#SPJ1

Scores at a local high school on the Algebra 1 Midterm are extremely skewed left with a mean of 65 and a standard deviation of 8. A guidance counselor takes a random sample of 10 students and calculates the mean score, x¯¯¯.



(a) Calculate the mean and standard deviation of the sampling distribution of x¯¯¯



(b) Would it be appropriate to use a normal distribution to model the sampling distribution? Justify your answer

Answers

The standard deviation of the sampling distribution is 2.53. It would not be appropriate to use a normal distribution to model the sampling distribution.

(a) When dealing with a sampling distribution, the mean of the sampling distribution (μ_x) is equal to the mean of the population (μ). In this case, the mean of the population is 65. Therefore, the mean of the sampling distribution is also 65.

To calculate the standard deviation of the sampling distribution (σ_x), you will use the following formula: σ_x = σ / √n, where σ is the standard deviation of the population and n is the sample size. In this case, the standard deviation of the population is 8 and the sample size is 10. So the standard deviation of the sampling distribution is:

σ_x = 8 / √10 ≈ 2.53

(b) The Central Limit Theorem states that the sampling distribution of the sample mean approaches a normal distribution as the sample size increases, regardless of the shape of the population distribution, provided that the population is not heavily skewed or has extreme outliers. Since the scores are extremely skewed left and the sample size is only 10, it would not be appropriate to use a normal distribution to model the sampling distribution. A larger sample size would be needed to use a normal distribution model.

More on sampling distribution: https://brainly.com/question/16311669

#SPJ11

A large corporation with monopolistic control in the marketplace has its average daily costs, in dollars, given by C =700 /x + 300x + x^2. The daily demand for x units of its product is given by p = 120,000 - 150 dollars. Find the quantity that gives maximum profit.

Answers

The quantity that gives maximum profit is approximately 111.55 units.

How to calculate the quantity that gives maximum profit

To find the quantity that gives maximum profit, we need to first find the revenue function and then the profit function.

The revenue function is given by:

R(x) = xp = x(120,000 - 150x)

The profit function is given by:

P(x) = R(x) - C(x) = x(120,000 - 150x) - (700/x + 300x + x²)

To find the quantity that gives maximum profit, we need to find the derivative of the profit function and set it equal to zero:

P'(x) = 120,000 - 300x - 700/x² - 2x

Setting P'(x) equal to zero and solving for x, we get:

120,000 - 300x - 700/x² - 2x = 0

Multiplying both sides by x^2, we get:

120,000x² - 300x³ - 700 - 2x³ = 0

Simplifying, we get:

300x³ + 2x³ - 120,000x² - 700 = 0

Dividing both sides by 2, we get:

151x³ - 60,000x² - 350 = 0

Using a graphing calculator or numerical methods, we can find that the real root of this equation is approximately x = 111.55.

Learn more about maximum profit at

https://brainly.com/question/29257255

#SPJ11

A parabola can be drawn given a focus of (−4,5) and a directrix of y=−9. what can be said about the parabola?

Answers

The parabola with a focus of (-4, 5) and a directrix of y = -9 is vertically oriented, opens upward, has a vertex at (-4, -2), and its equation is (x + 4)^2 = 28(y + 2).

1. The parabola is vertically oriented since the directrix is a horizontal line.
2. The vertex of the parabola is equidistant from the focus and the directrix. To find the vertex, we can calculate the midpoint between the focus and a point on the directrix with the same x-coordinate: (-4, -9 + (5 - (-9))/2) = (-4, -9 + 7) = (-4, -2).
3. The parabola opens upward because the focus is above the directrix.
4. The equation of the parabola can be found using the vertex form: (x - h)^2 = 4p(y - k), where (h, k) is the vertex and p is the distance between the vertex and the focus or directrix. In this case, (h, k) = (-4, -2), and p = (5 - (-2)) = 7. The equation is therefore (x + 4)^2 = 28(y + 2).

In summary, the parabola with a focus of (-4, 5) and a directrix of y = -9 is vertically oriented, opens upward, has a vertex at (-4, -2), and its equation is (x + 4)^2 = 28(y + 2).

learn more about "Parabola":-https://brainly.com/question/4061870

#SPJ11

In a circle, the acr length of an intercepted arc is ten inches. The radius of the circle measures 2 inches. What is the measure of the central angle that intercepts that acr?

Answers

The measure of the central angle that intercepts the arc of the circle is approximately 286.48 degrees.

A circle's circumference is comprised of arcs. In other words, if you draw any two locations on a circle's circumference, the curved line that joins them around the border of the circle is referred to as an arc.

The formula for the relationship between arc length and central angle is:

arc length = radius x central angle

We are given the arc length and radius, so we can solve for the central angle:

10 = 2 x central angle

Dividing both sides by 2, we get:

central angle = 10/2

= 5 radians

To convert from radians to degrees, we multiply by 180/π:

central angle = 5 x 180/π

≈ 286.48 degrees.

To know more about a circle, refer:

https://brainly.com/question/24176380

#SPJ4

12 less than the product of 3 and a number, x, is at most -18

Answers

The given inequality is 3x - 12 ≤ -18. To solve for x, we can add 12 to both sides of the inequality to obtain 3x ≤ -6. Then, dividing both sides of the inequality by 3 gives x ≤ -2. Therefore, any value of x less than or equal to -2 will satisfy the inequality.

In solving the inequality, we first used the addition property of inequalities to add 12 to both sides of the inequality. This property states that if a < b, then a + c < b + c, where c is any real number. By adding 12 to both sides, we were able to isolate the variable term on one side of the inequality.

Next, we used the division property of inequalities to divide both sides of the inequality by 3. This property states that if a < b and c > 0, then a/c < b/c. By dividing both sides of the inequality by 3, we were able to solve for x.

Finally, we found that any value of x less than or equal to -2 will satisfy the inequality. This means that the solution set for the inequality is {x | x ≤ -2}. We also verified that x = -2 is a valid solution to the inequality, which confirms our solution.

To know more about inequality refer here:

https://brainly.com/question/30231190#

#SPJ11

In triangle STU as equals 50 inches to equals 58 inches and you equals 27 inches find the measure of angle us to the nearest 10th of a degree

Answers

The measure of the angle ∠S, obtained using the law of cosines is about 59.4°

What is the law of cosines?

The law of cosines states that the square of the length of a side of a triangle is equivalent to the sum of the squares of the other two sides less the product of the length of the other two sides and the angle between them. Mathematically; a² = b² + c² - 2·b·c·cos(A)

Where;

a, b, and c = The length of the sides of the triangle

A = The angle between b and c

The lengths of the sides of the triangle, obtained from a similar triangle are;

s = 50 inches, t = 58 inches, and u = 27 inches

The measure of the angle ∠S is required

The angle ∠S is the angle facing TU or the side s

According to law of cosines, we get;

s² = t² + u² - 2·t·u·cos(∠S)

Therefore;

50² = 58² + 27² - 2 × 58 × 27 × cos(∠S)

2 × 58 × 27 × cos(∠S) = 58² + 27² - 50²

cos(∠S) = (58² + 27² - 50²)/(2 × 58 × 27)

∠S = cos((58² + 27² - 50²)/(2 × 58 × 27)) ≈ 59.4°

The measure of the angle ∠S is about 59.4°

Learn more on the law of cosines here: https://brainly.com/question/13409288

#SPJ4

4 Find the value xa where the function is discontinuousFor the point of discontinuity, give (a) f) if it exists. (b) lm (0) Im . () im 16), and (ej identity which conditions for continuity are not man -O (Use a coma o separate answers as needed Select the choice below and necessary, tal in the trawer box within your choice ОА ка) OB) is undefined (b) Select the choice below and necessary, tu in the answer box within your chale ΟΑ. lim) OB lim does not exist

Answers

So, the answer is:
(a) f(x) does not exist at xa = 16.
(b) lim f(x) as x approaches 16 does not exist.
(c) None of the conditions for continuity are met at xa = 16.

To find the value of xa where the function is discontinuous, we need to look for any points where the function is undefined or where the left and right limits of the function are not equal.
(a) From the given information, we know that the function is undefined at xa = 16. So, this is the point of discontinuity.
(b) To find the left and right limits at xa = 16, we need to approach the point from both sides of the function. So,
lim f(x) as x approaches 16 from the left (denoted as lim-) = Im = 0
lim f(x) as x approaches 16 from the right (denoted as lim+) = Im = 16
Since the left and right limits are not equal, the limit as x approaches 16 does not exist. So,
lim f(x) as x approaches 16 (denoted as lim) does not exist.
(c) To determine which conditions for continuity are not met, we need to check if the function satisfies the three conditions for continuity at xa = 16.
i) The function must be defined at xa = 16. Since the function is undefined at xa = 16, this condition is not met.
ii) The left and right limits of the function must exist and be equal at xa = 16. Since the left and right limits are not equal, this condition is not met.
iii) The value of the function at xa = 16 must be equal to the limit of the function at xa = 16. Since the limit does not exist, this condition is also not met.
Therefore, none of the conditions for continuity are met at xa = 16.
So, the answer is:
(a) f(x) does not exist at xa = 16.
(b) lim f(x) as x approaches 16 does not exist.
(c) None of the conditions for continuity are met at xa = 16.
Note: The terms "discontinuous" and "continuity" are used throughout the explanation to describe the concept and the point of interest. The term "function" refers to the given function that we are analyzing.

Learn more about continuity Visit: brainly.com/question/24637240

#SPJ11

Use the information in the table below to answer the following question. name of fund nav offer price upton group $18.47 $18.96 green energy $17.29 $18.01 tjh small-cap $18.43 $19.05 whi health $20.96 nl for which of the funds shown would you pay the most commission on the purchase of 100 shares? a. green energy b. tjh small-cap c. upton group d. whi health

Answers

WHI Health Fund pays the most commission on the purchase of 100 shares with a commission of $96.00. Thus, option d is correct.

Funds offer price for  Upton Group = $18.96 - $18.47

Funds offer price for Green Energy fund = $18.01 - $17.29

Funds offer price for TJH Small-Cap fund = $19.05 - $18.43

Funds offer price for WHI Health fund = $20.96 - $20.00

To calculate the commission on purchasing shares, we need to find the allowance between the price ranges and then multiply the value by 100.

For the Upton Group fund, Commission = (Offer price - NAV) * 100

= ($18.96 - $18.47) * 100

= $49.00

For the Green Energy fund, Commission = (Offer price - NAV) * 100

= ($18.01 - $17.29) * 100

= $72.00

For the TJH Small-Cap fund, Commission = (Offer price - NAV) * 100

= ($19.05 - $18.43) * 100

= $62.00

For the WHI Health fund, Commission = (Offer price - NAV) * 100

= ($20.96 - $20.00) * 100

= $96.00

Therefore, we can conclude that the WHI Health fund pays the most commission of $96.00.

To learn more about commission

https://brainly.com/question/12175457

#SPJ4

A line that passes through the point (x,y) with a y-intercept of b and a slope of m can be represented by the equation y = mx + b.
Joe drew a line on the coordinate plane that passes through the point (-10,52) and has a slope of -6.5. The y-intercept of the line is

Answers

The y-intercept of the line is -13.

To find the y-intercept of the line, we can use the slope-intercept form of the equation of a line: y = mx + b,

where m is the slope and b is the y-intercept.

Given that the line passes through the point (-10, 52) and has a slope of -6.5, we can substitute these values into the equation:

52 = -6.5(-10) + b

Simplifying the equation:

52 = 65 + b

To isolate b, we subtract 65 from both sides:

52 - 65 = b

Simplifying further:

b = -13

For similar question on y-intercept.

https://brainly.com/question/30239450

#SPJ11

. Use 3.14 for pi and round your answer to the nearest hundredth.
C= in
A = in^2

Answers

Answer:

A= 254.47 in

C= 56.55 in^2

Step-by-step explanation:

formula for area is πr^2 (radius is r)

circumference formula is πd or 2πr (diameter is d, radius is r)

I don't know what does C and A means but if A means area and C means circumference,

C = 56.52in

A = 254.34

Which statement is true considering a significance level of 5%?

A. The result is statistically significant, which implies that wearing a watch does not help people manage their time better.

B. The result is not statistically significant, which implies that this result could be due to random chance.

C. The result is statistically significant, which implies that wearing a watch helps people manage their time better.

D. The result is not statistically significant, which implies that wearing a watch does not help people manage their time better.

Answers

Given the scenerio in the picture about corn, the statement that is true looking at a significance level of 5% is The result is not statistically significant which implies that spraying the corn plants with the new type of fertilizer does increase the growth rate.

What is the does the 5% significance level mean in the context provided?

Looking at the statement "The result is not statistically significant,"this means that the p-value (probability value) of the test was greater than 0.05. It could have 0.15 oe 0.2.

When a test is greater than 0.05 or 5 % significance level, it shows that the what is happening to the corn (increase in growth rate) could have been as a result of chance alone.

Find more exercises on 5 % significance level;

https://brainly.com/question/24095750

#SPJ1

In 2010, Keenan paid $2,826 in federal income tax, which is 70% less than he paid in 2009. How much did he pay in 2009?

Answers

Based on the above, Keenan paid $9,420 in federal income tax in 2009.

What is the income tax?

Let X be the amount Keenan paid in taxes in 2009.

According to the problem, Keenan paid 70% less in 2010 than he did in 2009. This means that he paid only 30% of what he paid in 2009, since 100% - 70% = 30%.

We can express this mathematically as:

0.30X = 2,826

To solve for X, we can divide both sides of the equation by 0.30:

X = 2,826 ÷  0.30

X = 9,420

Therefore, Keenan paid $9,420 in federal income tax in 2009.

Learn more about income tax from

https://brainly.com/question/30157668

#SPJ1

what is 5 less than the square of a number in an algebraic expression

Answers

Answer:

let x be the no.

So, 5 less than the square of a number in an algebraic expression is:

x^2 - 5

What is 216 / 31? I keep on getting a decimal.

Answers

Answer:

6 30/31 as fraction and with Long Division its 6 with 30 Remainder

A city is planning a circular fountain, the depth of the fountain will be 3 feet in the volume will be 1800 feet to the third power, find the radius of the fountain, using the equation equals pi to the second power hhhh v is a volume in ours the radius and h is the depth round to the nearest whole number

Answers

The radius of the circular fountain is approximately 17 feet.

The formula for the volume of a circular fountain is given by V = πr^2h, where V is the volume, r is the radius, and h is the depth. In this case, we are given that the depth of the fountain is 3 feet and the volume is 1800 cubic feet. So we can plug in these values into the formula and solve for r as follows:

1800 = πr^2(3)

Simplifying this equation, we get:

r^2 = 600/π

Taking the square root of both sides, we get:

r = sqrt(600/π)

Using a calculator to approximate the value of sqrt(600/π), we get:

r ≈ 17

Therefore, the radius of the circular fountain is approximately 17 feet when rounded to the nearest whole number.

For more questions like Fountain click the link below:

https://brainly.com/question/22563529

#SPJ11

Ms. summers has 1/4 gallon of milk. she drinks 1/8 gallon of the milk and then splits the remaining milk equally between her two children. how much milk does ms. summers give each child? select the expression that could represent the situation

Answers

Ms. Summers gives each child 1/16 gallon of milk.

The amount of milk that Ms. Summers gives to each child can be represented by the following expression:

(1/4 gallon of milk - 1/8 gallon of milk) / 2

This expression represents the amount of milk that remains after Ms. Summers drinks 1/8 gallon of milk, divided equally between her two children.

Simplifying the expression, we get:

(2/8 - 1/8) / 2 = 1/8 / 2 = 1/16

Therefore, Ms. Summers gives each child 1/16 gallon of milk.

To know more about milk refer here

https://brainly.com/question/222134#

#SPJ11

At the neighborhood grocery, 5 pounds of chicken thighs cost $23.75. Riley spent $15.96 on chicken thighs. How many pounds of chicken thighs did she buy, to the nearest hundredth of a pound? 2​

Answers

Using the given information, Riley bought 3.36 pounds of chicken thighs

Calculating the pounds of chicken bought

From the question, we are to calculate the number of pounds of chicken thighs that Riley bought

We can use proportionality to find how many pounds of chicken thighs Riley bought.

If 5 pounds of chicken thighs cost $23.75, then we can write the following proportion:

Cost/Weight = $23.75/5 lb

We can use this proportion to find the cost per pound of chicken thighs:

That is,

Cost/Weight = $23.75/5 lb = $4.75/lb

Now we can use this rate to find how many pounds of chicken thighs Riley bought:

Cost of chicken thighs bought = $15.96

Weight of chicken thighs = Cost of chicken thighs / Cost per pound of chicken thighs

Weight of chicken thighs bought = $15.96 / $4.75/lb ≈ 3.36 lb

Hence, Riley bought 3.36 pounds of chicken thighs.

Learn more on Calculating the pounds of chicken bought here: https://brainly.com/question/25713291

#SPJ1

Katya collects badges from the National Parks Junior Rangers program.


She wants to know how many cases she needs to display her collection. She can fit 8 badges in one case.


There are 207 National Parks that participate in the Junior Ranger badge program.


She does not have the badges for 165 parks. Which equation should she use first to solve this problem?


A)b=207-165


B)207+165


C)42 divided by 8


D)42*8(42 multiplied by 8)

Answers

After solving equation, Katya needs 5 or 6 display cases to display all of her badges, depending on how she chooses to group them.

The number of badges Katya has is not explicitly given in the problem. Therefore, we need to first find the total number of badges Katya has earned by subtracting the number of parks for which she doesn't have badges from the total number of participating parks.

The equation to use for this is:

b = 207 - 165

where b is the total number of badges Katya has earned.

Option A is the correct equation, which simplifies to:

b = 42

This means Katya has earned 42 badges.

Next, we need to find out how many display cases Katya needs. We know that she can fit 8 badges in one case, so we can use division to find the number of cases needed:

c = b / 8

where c is the number of cases needed.

Substituting the value of b from the previous equation, we get:

c = 42 / 8

Option C is the correct equation, which simplifies to:

c = 5.25

This means that Katya needs 5 or 6 display cases to display all of her badges, depending on how she chooses to group them.

To know more about equation, visit:

https://brainly.com/question/29657983#

#SPJ11

A sports arena has 40 soda vendors. Each of whom sells 200 sodas per event. Management estimates that for each additional vendor, the yield per vendor decreases by 4. How many additional vendors should management hire to maximize the number of sodas sold.

Answers

Management should hire 25 additional vendors to maximize the number of sodas sold.

Let x be the number of additional vendors that management hires. Then the total number of vendors is 40 + x, and the yield per vendor is 200 - 4x (since the yield decreases by 4 for each additional vendor).

The total number of sodas sold is the product of the number of vendors and the yield per vendor:

Total sodas sold = (40 + x) * (200 - 4x)

To maximize the number of sodas sold, we take the derivative of this expression with respect to x and set it equal to zero:

d/dx [(40 + x) * (200 - 4x)] =

Expanding and simplifying, we get:

-8x² + 120x + 8000 = 0

Dividing both sides by -8, we get:

x² - 15x - 1000 = 0

Using the quadratic formula, we solve for x:

x = (15 ± sqrt(15² + 411000)) / 2

x = (15 ± 35) / 2

x = -10 or x = 25

Since we can't hire a negative number of vendors, the only sensible solution is x = 25. Therefore, management should hire 25 additional vendors to maximize the number of sodas sold.

To learn more about quadratic formula visit: https://brainly.com/question/2615966

#SPJ11

Dai orders milk with her meal. The server asks her if she wants regular or chocolate. Dai can choose from skim, 2%, or whole, and from small, medium, or large. If all of the choices are equally likely to be ordered, what is the probability that Dai orders a regular, medium milk? Write a whole number or fractions

Answers

The probability of Dai ordering a regular, medium milk is 1/18.

What is the probability of an event? Calculate the total number of possible milk orders.

There are 2 types of milk (regular and chocolate), and 3 sizes (small, medium, and large), and 3 levels of fat content (skim, 2%, and whole). So the total number of possible milk orders is:

2 (types of milk) x 3 (sizes) x 3 (fat content) = 18

Calculate the number of ways Dai can order a regular, medium milk.

Dai needs to choose regular milk and medium size, so there is only one way she can order this combination.

Calculate the probability of Dai ordering a regular, medium milk.

The probability of Dai ordering a regular, medium milk is the number of ways she can order a regular, medium milk divided by the total number of possible milk orders:

1 (number of ways to order a regular, medium milk) / 18 (total number of possible milk orders) = 1/18

So the probability that Dai orders a regular, medium milk is 1/18 or approximately 0.056 (rounded to three decimal places).

Learn more about Probability.

brainly.com/question/30034780

#SPJ11

The length of the hypotenuse in a °45 degrees-°45 degrees-°90 degrees triangle is 5 square root of 2. What are the sine and secant ratios for a °45 angle?

Answers

The secant of a °45 angle is defined as the ratio of the length of the hypotenuse to the length of the adjacent side. In this case, the adjacent side is also a leg of length 5, so:

secant(45) = hypotenuse/adjacent = 5√2/5 = √2

What is the secant function?

The secant function, denoted as sec(x), is a trigonometric function that is defined as the reciprocal of the cosine function, cos(x).

In other words,

sec(x) = 1 / cos(x)

The secant function is defined for all values of x except for those where the cosine function is equal to zero, which corresponds to the values x = (2n+1)π/2 where n is an integer. At these points, the secant function is undefined.

According to the given function

In a °45-°45-°90 triangle, the two legs are congruent, so if the length of the hypotenuse is 5√2, then each leg has a length of:

leg = hypotenuse/√2 = (5√2)/√2 = 5

The sine of a °45 angle is defined as the ratio of the length of the side opposite the angle to the length of the hypotenuse. In this case, the side opposite the angle is a leg of length 5, so:

sine(45) = opposite/hypotenuse = 5/5√2 = 1/√2 = √2/2

The secant of a °45 angle is defined as the ratio of the length of the hypotenuse to the length of the adjacent side. In this case, the adjacent side is also a leg of length 5, so:

secant(45) = hypotenuse/adjacent = 5√2/5 = √2

To know more about the secant function visit:

brainly.com/question/16230804

#SPJ1

Use the Picard-Lindeloef iteration to find the first few elements of a sequence {yn}n=0 of approximate solutions to the initial value problem y(t) = 5y(t)+1, y(0) = 0

Answers

To use the Picard-Lindelöf iteration to find a sequence of approximate solutions to the initial value problem y'(t) = 5y(t) + 1, y(0) = 0, we start with the initial approximation y_0(t) = 0. Then, for each n ≥ 0, we define y_{n+1}(t) to be the solution to the initial value problem y'(t) = 5y_n(t) + 1, y_n(0) = 0. In other words, we plug the previous approximation y_n into the right-hand side of the differential equation and solve for y_{n+1}.Using this procedure, we can find the first few elements of the sequence {y_n} as follows:y_0(t) = 0y_1(t) = ∫ (5y_0(t) + 1) dt = ∫ 1 dt = ty_2(t) = ∫ (5y_1(t) + 1) dt = ∫ (5t + 1) dt = (5/2)t^2 + ty_3(t) = ∫ (5y_2(t) + 1) dt = ∫ (5(5/2)t^2 + 5t + 1) dt = (25/6)t^3 + (5/2)t^2 + tTherefore, the first few elements of the sequence {y_n} are y_0(t) = 0, y_1(t) = t, y_2(t) = (5/2)t^2 + t, and y_3(t) = (25/6)t^3 + (5/2)t^2 + t.

For more similar questions on topic Differential equations.

https://brainly.com/question/14620493?referrer=searchResults

#SPJ11

To use the Picard-Lindelöf iteration method to find the first few elements of a sequence {y_n} of approximate solutions to the initial value problem y'(t) = 5y(t) + 1, y(0) = 0, we first set up the integral equation for the iteration:

y_n+1(t) = y(0) + ∫[5y_n(s) + 1] ds from 0 to t

Since y(0) = 0, the equation becomes:

y_n+1(t) = ∫[5y_n(s) + 1] ds from 0 to t

Now, let's calculate the first few approximations:

1. For n = 0, we start with y_0(t) = 0:
y_1(t) = ∫[5(0) + 1] ds from 0 to t = ∫1 ds from 0 to t = s evaluated from 0 to t = t

2. For n = 1, use y_1(t) = t:
y_2(t) = ∫[5t + 1] ds from 0 to t = 5/2 s^2 + s evaluated from 0 to t = 5/2 t^2 + t

3. For n = 2, use y_2(t) = 5/2 t^2 + t:
y_3(t) = ∫[5(5/2 t^2 + t) + 1] ds from 0 to t = ∫(25/2 t^2 + 5t + 1) ds from 0 to t = 25/6 t^3 + 5/2 t^2 + t

These are the first few elements of the sequence {y_n} of approximate solutions to the initial value problem using the Picard-Lindelöf iteration method.

To learn more about sequence, refer below:

https://brainly.com/question/30262438

#SPJ11

A book club of 7 members meet at a local coffee shop. One week, 5 of the
members ordered a small cup of coffee and a muffin. The other 2 members
ordered a small cup of coffee and a piece of banana bread. The cost of a muffin,
including tax, is $3.51. The cost of piece of banana bread is $2. 16 more than
the cup of coffee. The total bill for the book club was $48. 60.

Answers

The cost of a small cup of coffee is $2.97, and the cost of a piece of banana bread is $5.13.

How to solve

Let x represent the cost of a small coffee and y represent the cost of a piece of banana bread. We know:

Cost of muffin: $3.51

y = x + $2.16

5(x + $3.51) + 2(x + y) = $48.60

Substitute y with x + $2.16:

5(x + $3.51) + 2(x + (x + $2.16)) = $48.60

Solve for x:

9x + $21.87 = $48.60

9x = $26.73

x = $2.97

Find y:

y = x + $2.16

y = $2.97 + $2.16

y = $5.13

A slice of banana bread costs $5.13, while a small coffee costs $2.97.

Read more about algebra here:

https://brainly.com/question/432678

#SPJ1

What is the variance of the following set of data?


4, 44, 404, 244, 4, 74, 84, 64

Answers

The variance of the given data set is 18603.39.

To find the variance of the given data set {4, 44, 404, 244, 4, 74, 84, 64}, follow these steps:

Step 1: First, we need to find the mean of the data set:

Mean = (4 + 44 + 404 + 244 + 4 + 74 + 84 + 64) / 8 = 120.5

Step 2: Next, we calculate the deviation of each data point from the mean:

(4 - 120.5) = -116.5

(44 - 120.5) = -76.5

(404 - 120.5) = 283.5

(244 - 120.5) = 123.5

(4 - 120.5) = -116.5

(74 - 120.5) = -46.5

(84 - 120.5) = -36.5

(64 - 120.5) = -56.5

Step 3: Now, we square each deviation:

[tex](-116.5)^2 = 13556.25\\(-76.5)^2 = 5852.25\\(283.5)^2 = 80322.25\\(123.5)^2 = 15252.25\\(-116.5)^2 = 13556.25\\(-46.5)^2 = 2162.25 \\(-36.5)^2 = 1332.25\\(-56.5)^2 = 3192.25[/tex](-116.5)^2 = 13556.25

Step 4: We add up all the squared deviations:

13556.25 + 5852.25 + 80322.25 + 15252.25 + 13556.25 + 2162.25 + 1332.25 + 3192.25 = 130223.75

Step 5: We divide the sum of the squared deviations by the number of data points minus 1 to get the variance:

Variance = 130223.75 / 7 = 18603.39 (rounded to two decimal places)

Therefore, the variance of the data set is 18603.39.

To know more about Variance refer here:

https://brainly.com/question/14116780

#SPJ11


PLEASE HELP!! WILL GIVE BRAINLIEST!!! FIRST ANSWER GETS IT!!

The graph of f(x) and table for g(x)= f(kx) are given.

A coordinate plane with a quadratic function labeled f of x that passes through the points negative 2 comma 4 and negative 1 comma one and vertex 0 comma 0 and 1 comma 1 and 2 comma 4
x g(x)
−2 64
−1 16
0 0
1 16
2 64

What is the value of k?
k = -4
k = 4
k = -1/4

Answers

Answer:

k = 1

Step-by-step explanation:

We can use the table for g(x) = f(kx) to find the value of k.

Notice that when x = -2, we have g(-2) = f(k(-2)) = f(-2k) = 64. Similarly, when x = 2, we have g(2) = f(k(2)) = f(2k) = 64.

Using the fact that f(x) is a quadratic function, we can see that its axis of symmetry passes through the vertex at (0, 0), which means that the x-coordinate of the vertex is 0. This tells us that the coefficient of the x term in f(x) is 0, so the function can be written as f(x) = ax^2 + bx + c, where a is not equal to 0.

Using the points (−2,4), (−1,1), (0,0), (1,1), and (2,4), we can write a system of equations to solve for a, b, and c:

a(-2)^2 + b(-2) + c = 4

a(-1)^2 + b(-1) + c = 1

a(0)^2 + b(0) + c = 0

a(1)^2 + b(1) + c = 1

a(2)^2 + b(2) + c = 4

Simplifying and rearranging, we get:

4a - 2b + c = 4

a - b + c = 1

c = 0

a + b + c = 1

4a + 2b + c = 4

Substituting c = 0 into the system, we get:

4a - 2b = 4

a - b = 1

a + b = 1

4a + 2b = 4

Solving this system of equations, we get a = 1, b = -1, and c = 0.

Substituting these values into g(x) = f(kx), we get:

g(x) = f(kx) = x^2 - x

Substituting the values from the table into this equation, we get:

g(-2) = 4 = (-2)^2 - (-2) = 4k

g(2) = 4 = (2)^2 - (2) = 4k

Solving for k, we get k = 1 or k = -1/4.

However, we need to check which value of k satisfies all the points in between -2 and 2, so we can check g(-1) = 1 = (-1)^2 - (-1) = k, and g(1) = 1 = (1)^2 - (1) = k.

Thus, the value of k that satisfies all the points is k = 1, and therefore the answer is:

k = 1

We can use the information given to find the value of k.

Since the vertex of the quadratic function f(x) is at (0,0), we know that the equation for f(x) is in the form of f(x) = ax^2 for some constant a.

Using the point (-2, 4) on the graph of f(x), we can set up the equation 4 = 4a, which gives us a = 1.

So, the equation for f(x) is f(x) = x^2.

Now, we can use the table for g(x) = f(kx) to find the value of k.

When x = -2, we have g(-2) = f(k(-2)) = f(-2k) = 4k^2.

Similarly, when x = 2, we have g(2) = f(k(2)) = f(2k) = 4k^2.

We also know that g(0) = f(k(0)) = f(0) = 0, and g(-1) = f(k(-1)) = f(-k) = k^2 and g(1) = f(k(1)) = f(k) = k^2.

Using the values from the table, we can set up the following system of equations:

4k^2 = 64

k^2 = 16

0 = 0

k^2 = 16

The only solution that works for all of these equations is k = 4 or k = -4.

Therefore, the value of k is either k = 4 or k = -4.

Determine Whether The Series Is Convergent Or Divergent. Σ ^n√14

Answers

Based on the Root Test, the series Σ^n√14 is convergent.

Hi! To determine if the series Σ^n√14 is convergent or divergent, we need to analyze the terms involved. The series can be written as:

Σ (n√14)

This is a sum of terms, where each term is the n-th root of 14, and we want to find out if the sum converges or diverges as n goes to infinity.

In this case, the series is a type of p-series, where the terms follow the general form of 1/n^p. To be a convergent p-series, p must be greater than 1. Here, the terms are in the form of 14^(1/n), which can be rewritten as (14^(1))^(-n) or 14^(-n). This is not a p-series, as the exponent is not in the form of 1/n^p.

To further analyze the series, we can use the Divergence Test. If the limit of the terms as n goes to infinity is not equal to zero, then the series is divergent. So, let's find the limit:

lim (n → ∞) (14^(-n))

As n approaches infinity, the exponent -n becomes increasingly negative, and 14^(-n) approaches 0. However, the Divergence Test is inconclusive in this case, as it only confirms divergence if the limit is not equal to zero.

To determine convergence or divergence, we can use the Root Test. The Root Test states that if the limit of the n-th root of the absolute value of the terms as n goes to infinity is less than 1, then the series converges. Let's find the limit:

lim (n → ∞) |(14^(-n))|^(1/n)

This simplifies to:

lim (n → ∞) 14^(-1)

Since 14^(-1) is a constant value less than 1, the limit is less than 1.

Thus, based on the Root Test, the series Σ^n√14 is convergent.

To learn more about root test, refer below:

https://brainly.com/question/30906556

#SPJ11

Other Questions
17. The Amethyst Woodstar species ofhummingbird beats its wings about80 times per second. If there are60 seconds in a minute, about howmany times does the AmethystWoodstar beat its wings in9 minutes?A 12,600B 37,800C50,400D 43,200 you are trying to evaluate expansion plans for heb that will be financed with no debt. for this project the discount rate is 9%. your cash flows will be $1 m, $3 m, and $4 m for the first 3 years and grow at 3% from then on. if this expansion costs $50 m, what is the npv? Identify and compare the differences between the leadership styles of Martin and his predecessor Otto Why did the US believe Europe was fertile for communism? The floors and walls of gerald's attic form an equilateral triangle what is the approximate height h of the attic? Carmichael Cleaners needs a new steam finishing machine that costs $100,000. The company is evaluating whether it should lease or purchase the machine. The equipment falls into the MACRS 3-year class, and it would be used for 3 years and then sold, because the firm plans to move to a new facility at that time. The estimated value of the equipment after 3 years is $30,000. A maintenance contract on the equipment would cost $3,000 per year, payable at the beginning of each year. Alternatively, the firm could lease the equipment for 3 years for a lease payment of $29,000 per year, payable at the beginning of each year. The lease would include maintenance. Due to special circumstances, the firm is in the 20% tax bracket, and it could obtain a 3-year simple interest loan, interest payable at the end of the year, to purchase the equipment at a before-tax cost of 10%. If there is a positive Net Advantage to Leasing the firm will lease the equipment. Otherwise, it will buy it. What is the NAL? (Note: Assume MACRS rates for Years 1 to 4 are 0. 3333, 0. 4445, 0. 1481, and 0. 741. ) Suppose 45% of all students at aiden's school brought a can of food to contribute to a canned food drive. aiden picks a representative sample of 25 students and determines the samples percentage he expects the percentage for this sample will be 45% do you agree? explain your reasoning theta is a right angled triangle theta = tan-1(0.43) work out the value of theta give your answer to 1 decimal place Matthew is working two summer jobs, making $9 per hour babysitting and making $13 per hour landscaping. In a given week, he can work a maximum of 16 total hours and must earn at least $180. If Matthew worked 3 hours babysitting, determine the maximum number of whole hours landscaping that he can work and still meet his requirements. If there are no possible solutions, submit an empty answer. A decomposition of hydrogen peroxide into water and oxygen gas is an exothermic reaction. If the temperature is initially 28 C, what would you expect to see happen to the final temperature?Explain what is happening in terms of energy of the system and the surroundings. FAST FIRST ONE GETS BRAINLIEST Why was the fact that radio masked the race of the performers helpful to many performers?Select one:a.Most musicians did not want anyone to know what race they were.b.Many radio stations were owned and run by minority business owners.c.After the rise of radio, there was no discrimination in the music industry.d.It allowed access to audiences that would have rejected them based on race. find the value of x in the cube A triangular roof is built so that its height is half its base. If the base of the roof is 32 feet long, what is the area of the roof? 4. Lana has a bag of marbles. The probability of picking a striped marble is 8%. If Lana picks a marble and then replaces it 320 times, predict about how many times she would pick a marble that is not striped. If a solid mixture of the three aromatic compounds shown below is placed in 3 m hcl, which is likely to dissolve?. process for citizens to directly approve laws or state constitutional amendments are called x + 5 < 7 x + 5 -? ? 7-?x ?? -5 - 4 - 3 - 2 - 1 0 1 2 3 4 5 Graph the solution after you get your answer ). Brooklane Ltd. Has recently appointed a new CEO of the company. The CEO has several new ideas for the organisation. One idea, which he wants to explore is the elimination of annual budgets. You are working as a management team member in this company and CEO has asked you to write a report which investigates this idea. Questiona)Write a report for the CEO setting out the advantages and limitations of an annual budgeting system. Use academic sources to support your answer. Part (two). The following information relates to the wards in the cardiology department of Good Health Hospital for one accounting periodWards1,2,3Number of Beds12,10,20Total number of Patients48,32,108Total ward staff (excluding consultants)8,8,12Ward staff full-time equivalent5,4,6Variable operating costs for period58,000,66,000,98,000Operating fixed costs85,000,67,000,88,000General overhead costs40,000,40,000,40,000General overhead costs are made up of recharges from support services. The total cost of 120,000 is split equally between the three wards. The hospital manager is concerned about the high costs of running the three heart-care wards. She has asked you to investigate their performance. Questionb)Write a report which investigates the costs of operating the three hospital wards. Your report should include appropriate measures of performance for each ward and you should draw conclusions from the costs. Calculate necessary calculations to support your conclusions. [Hint: Calculations include: Patient (turnover) per bed, FTE Staff per bed & per patients, Variable Cost per bed & per patient and Fixed cost per bed & per patient. ] Please help with trigonometry Find the sum of the arithmetic sequence: 7+11+15+19+...+83+87.